Đến nội dung

Hình ảnh

Các bài toán đa thức qua các kì thi HSG

- - - - -

  • Please log in to reply
Chủ đề này có 48 trả lời

#1
alex_hoang

alex_hoang

    Thượng úy

  • Hiệp sỹ
  • 1152 Bài viết

Các bài toán đa thức qua các kì thi HSG

Đa thức là phần rất hay gặp trong các kì thi HSG trong topic này chúng ta sẽ cùng trao đổi về nó

Bài1: Cho $f(x)$ là một đa thức có bậc $n$.CMR $f(x)$ chia hết cho $f'(x)$ khi và chỉ khi $f(x)$ có dạng ${a_n}{\left( {x - {x_0}} \right)^n}$ với giá trị $x_0$ nào đó


Bài viết đã được chỉnh sửa nội dung bởi alex_hoang: 23-11-2011 - 21:50

alex_hoang


HẸN NGÀY TRỞ LẠI VMF THÂN MẾN

http://www.scribd.co...oi-Ban-Cung-The

#2
alex_hoang

alex_hoang

    Thượng úy

  • Hiệp sỹ
  • 1152 Bài viết
Bài 2:Tìm đa thức $P(x)$ thỏa mãn
\[P(P(x)) + 1 = {\left[ {{P^2}(x) + 2P(x) + {{\left( {{x^2} + 3x + 1} \right)}^2}} \right]^2}\]
alex_hoang


HẸN NGÀY TRỞ LẠI VMF THÂN MẾN

http://www.scribd.co...oi-Ban-Cung-The

#3
alex_hoang

alex_hoang

    Thượng úy

  • Hiệp sỹ
  • 1152 Bài viết

Bài 2:Tìm đa thức $P(x)$ thỏa mãn
\[P(P(x)) + 1 = {\left[ {{P^2}(x) + 2P(x) + {{\left( {{x^2} + 3x + 1} \right)}^2}} \right]^2};(1)\]

Giải:
Ta có $P(x)=0$ không thỏa mãn,vậy $P(x)$ có dạng
\[P(x) = {a_n}{x^n} + {a_{n - 1}}{x^{n - 1}} + ... + {a_1}x + {a_0};({a_n} \ne 0)\]
Khi khai triển hai vế của $(1)$ thì số hạng lớn nhất của $P(P(x))+1$ là:
\[{a_n}{({a_n}{x^n})^n} = a_n^{n + 1}{x^{{n^2}}}\]
Còn số hạng lớn nhất của${\left[ {{P^2}(x) + 2P(x) + {{\left( {{x^2} + 3x + 1} \right)}^2}} \right]^2}$ là
$\left[ {\begin{array}{{{\left( {{{\left( {{a_n}{x^n}} \right)}^2}} \right)}^2} = a_n^4{x^{4n}};n > 2}\\{{{\left( {{{\left( {{a_n}{x^n}} \right)}^2} + {x^4}} \right)}^2} = {{(a_n^2 + 1)}^2}{x^8};n = 2}\\{{x^8};n < 2}\end{array}} \right.$
Nếu $n \le 2$ thì
\[{x^{{n^2}}} = {x^8} \Rightarrow {n^2} = 8\]
Điều trên là vô lý vì n là số tự nhiên
Vậy $n>2$ và \[a_n^{n + 1}{x^{{n^2}}} = a_n^4{x^{4n}} \Rightarrow n = 4;{a_n} = 1\]
Do vậy đa thức cần tìm có dạng
\[P(x) = {x^4} + {a_3}{x^3} + {a_2}{x^2} + {a_1}x + {a_0}\]
Ta đặt
\[G(x) = P(x) - {\left( {{x^2} + 3x + 1} \right)^2} + 1\]
Vậy thì bây giờ $(1)$ tương đương với
\[P(P(x)) + 1 = {\left[ {{P^2}(x) + 3P(x) + 1 - G(x)} \right]^2}\]
\[ \Leftrightarrow P(P(x)) - {\left( {{P^2}(x) + 3P(x) + 1} \right)^2} + 1 = G(x)\left[ {G(x) - 2({P^2}(x) + 3P(x) + 1)} \right]\]
\[ \Leftrightarrow G(P(x)) = G(x)\left[ {G(x) - 2({P^2}(x) + 3P(x) + 1)} \right]\]
Nếu mà $G(x)$ khác $0$ ta đặt $degG(x)=k;(k\le3)$ nên ta có
$4k=k+8$
Vô lí vì k là số tự nhiên Vậy $G(x)=0$ từ đó ta có $P(x)=x(x+1)(x+2)(x+3)$

Bài viết đã được chỉnh sửa nội dung bởi alex_hoang: 25-11-2011 - 22:13

alex_hoang


HẸN NGÀY TRỞ LẠI VMF THÂN MẾN

http://www.scribd.co...oi-Ban-Cung-The

#4
alex_hoang

alex_hoang

    Thượng úy

  • Hiệp sỹ
  • 1152 Bài viết
Bài 3:Cho đa thức bậc $n$ có $n$ nghiệm phân biệt là$x_1,x_2,...x_n$.Chứng minh rằng
a)\[\frac{{P"({x_1})}}{{P'({x_1})}} + \frac{{P"({x_2})}}{{P'({x_2})}} + ... + \frac{{P"({x_n})}}{{P'({x_n})}} = 0\]
b)\[\frac{1}{{P'({x_1})}} + \frac{1}{{P'({x_2})}} + .... + \frac{1}{{P'({x_n})}} = 0\]

Bài viết đã được chỉnh sửa nội dung bởi alex_hoang: 25-11-2011 - 22:18

alex_hoang


HẸN NGÀY TRỞ LẠI VMF THÂN MẾN

http://www.scribd.co...oi-Ban-Cung-The

#5
alex_hoang

alex_hoang

    Thượng úy

  • Hiệp sỹ
  • 1152 Bài viết
Bài 4:Xác định tất cả các đa thức (có hệ số thực)$P,Q$ và $R$ thỏa mãn phương trình
\[\sqrt {P(x)} - \sqrt {Q(x)} = R(x)\]
Với mọi số thực $x$
alex_hoang


HẸN NGÀY TRỞ LẠI VMF THÂN MẾN

http://www.scribd.co...oi-Ban-Cung-The

#6
Crystal

Crystal

    ANGRY BIRDS

  • Hiệp sỹ
  • 5534 Bài viết

Bài 3:Cho đa thức bậc $n$ có $n$ nghiệm phân biệt là$x_1,x_2,...x_n$.Chứng minh rằng
a)\[\dfrac{{P"({x_1})}}{{P'({x_1})}} + \dfrac{{P"({x_2})}}{{P'({x_2})}} + ... + \dfrac{{P"({x_n})}}{{P'({x_n})}} = 0\]

Câu a:

Xét $P\left( x \right) = \left( {x - {x_1}} \right)\left( {x - {x_2}} \right)...\left( {x - {x_n}} \right)$, giả sử ${x_1} < {x_2} < ... < {x_n}$
Ta có: $$P'\left( x \right) = P\left( x \right)\left( {\dfrac{1}{{x - {x_1}}} + \dfrac{1}{{x - {x_2}}} + ... + \dfrac{1}{{x - {x_n}}}} \right) = P\left( x \right)\sum\limits_{i = 1}^n {\dfrac{1}{{x - {x_i}}}} \,\,\,\,\,\left( 1 \right)$$
Do $P\left( {{x_i}} \right) = 0,i = \overline {1,n} $ nên theo định lí Rolle tồn tại ${c_1},{c_2},...,{c_{n - 1}};\,\,{x_1} < {c_1} < {x_2} < {c_2} < ... < {c_{n - 1}} < {x_n}$ sao cho $P'\left( {{c_i}} \right) = 0,i = \overline {1,n - 1} \,\,\,\,\,\,\,\,\,\,\left( 2 \right)$
Lại có: $$P''\left( x \right) = P'\left( x \right)\left( {\dfrac{1}{{x - {c_1}}} + \dfrac{1}{{x - {c_2}}} + ... + \dfrac{1}{{x - {c_{n - 1}}}}} \right) = P'\left( x \right)\sum\limits_{i = 1}^{n - 1} {\dfrac{1}{{x - {c_i}}}} \,\,\,\,\,\left( 3 \right)$$
Từ (1) và (2) suy ra:$$\left\{ \begin{array}{l}
P'\left( {{c_1}} \right) = P\left( {{c_1}} \right)\left( {\dfrac{1}{{{c_1} - {x_1}}} + \dfrac{1}{{{c_1} - {x_2}}} + ... + \dfrac{1}{{{c_1} - {x_n}}}} \right) = P\left( {{c_1}} \right)\sum\limits_{i = 1}^n {\dfrac{1}{{{c_1} - {x_i}}} = 0} \\
P'\left( {{c_2}} \right) = P\left( {{c_2}} \right)\left( {\dfrac{1}{{{c_2} - {x_1}}} + \dfrac{1}{{{c_2} - {x_2}}} + ... + \dfrac{1}{{{c_2} - {x_n}}}} \right) = P\left( {{c_2}} \right)\sum\limits_{i = 1}^n {\dfrac{1}{{{c_2} - {x_i}}} = 0} \\
...............\\
P'\left( {{c_{n - 1}}} \right) = P\left( {{c_{n - 1}}} \right)\left( {\dfrac{1}{{{c_{n - 1}} - {x_1}}} + \dfrac{1}{{{c_{n - 1}} - {x_2}}} + ... + \dfrac{1}{{{c_{n - 1}} - {x_n}}}} \right) = P\left( {{c_{n - 1}}} \right)\sum\limits_{i = 1}^n {\dfrac{1}{{{c_{n - 1}} - {x_i}}} = 0}
\end{array} \right.$$
Do $P\left( {{c_i}} \right) \ne 0,i = \overline {1,n - 1} $ nên ta có:$$\left\{ \begin{array}{l}
\dfrac{1}{{{c_1} - {x_1}}} + \dfrac{1}{{{c_1} - {x_2}}} + ... + \dfrac{1}{{{c_1} - {x_n}}} = \sum\limits_{i = 1}^n {\dfrac{1}{{{c_1} - {x_i}}} = 0} \\
\dfrac{1}{{{c_2} - {x_1}}} + \dfrac{1}{{{c_2} - {x_2}}} + ... + \dfrac{1}{{{c_2} - {x_n}}} = \sum\limits_{i = 1}^n {\dfrac{1}{{{c_2} - {x_i}}} = 0} \\
..............\\
\dfrac{1}{{{c_{n - 1}} - {x_1}}} + \dfrac{1}{{{c_{n - 1}} - {x_2}}} + ... + \dfrac{1}{{{c_{n - 1}} - {x_n}}} = \sum\limits_{i = 1}^n {\dfrac{1}{{{c_{n - 1}} - {x_i}}} = 0}
\end{array} \right.$$
Suy ra: $$\sum\limits_{i = 1}^n {\dfrac{{P''\left( {{x_i}} \right)}}{{P'\left( {{x_i}} \right)}} = } \sum\limits_{i = 1}^n {\dfrac{1}{{{c_1} - {x_i}}} + \sum\limits_{i = 1}^n {\dfrac{1}{{{c_2} - {x_i}}} + \sum\limits_{i = 1}^n {\dfrac{1}{{{c_{n - 1}} - {x_i}}} = 0} } } $$
Ta có đpcm.

#7
Crystal

Crystal

    ANGRY BIRDS

  • Hiệp sỹ
  • 5534 Bài viết

Bài 3:Cho đa thức bậc $n$ có $n$ nghiệm phân biệt là$x_1,x_2,...x_n$.Chứng minh rằng
b)\[\dfrac{1}{{P'({x_1})}} + \dfrac{1}{{P'({x_2})}} + .... + \dfrac{1}{{P'({x_n})}} = 0\]

Xét phân tích: $$P\left( x \right) = \left( {x - {x_1}} \right)\left( {x - {x_2}} \right)...\left( {x - {x_n}} \right) = \prod\limits_{i = 1}^n {\left( {x - {x_i}} \right)} $$
Đặt: $${P_i}\left( x \right) = \prod\limits_{j = 1,j \ne i}^n {\left( {x - {x_j}} \right)} \Rightarrow P'\left( x \right) = \sum\limits_{i = 1}^n {{P_i}\left( x \right)} $$
Ngoài ra: $${P_i}\left( {{x_j}} \right) = 0,\forall j \ne i;\,\,{P_i}\left( {{x_i}} \right) \ne 0,\forall i = \overline {1,n} \Rightarrow P'\left( {{x_i}} \right) = {P_i}\left( {{x_i}} \right),\forall i = \overline {1,n} $$
Xét đa thức $Q\left( x \right) = \sum\limits_{i = 1}^n {\dfrac{{{P_i}\left( x \right)}}{{P'\left( {{x_i}} \right)}} - 1} $ có bậc không vượt quá $n-1$. Ta có $\forall i = \overline {1,n} \Rightarrow Q\left( {{x_i}} \right) = 0$

Suy ra đa thức này có $n$ nghiệm thực phân biệt, tức $Q\left( x \right) \equiv 0$ và hệ số cao nhất của $Q\left( x \right)$ cũng bằng $0$.
Do đó: $$\dfrac{1}{{P'({x_1})}} + \dfrac{1}{{P'({x_2})}} + .... + \dfrac{1}{{P'({x_n})}} = 0$$
Ta có đpcm.

#8
Crystal

Crystal

    ANGRY BIRDS

  • Hiệp sỹ
  • 5534 Bài viết
Ủng hộ Hoàng một bài.

Bài 5: Cho $P\left( x \right)$ là một đa thức bậc $n \ge 1$ với hệ số thực và có $n$ nghiệm thực. Chứng minh rằng:
$$\left ( n-1 \right ){P}'^{2}\left ( x \right )\geq nP\left ( x \right ){P}''\left ( x \right ),\; \forall x\in \mathbb{R}$$

#9
alex_hoang

alex_hoang

    Thượng úy

  • Hiệp sỹ
  • 1152 Bài viết

Ủng hộ Hoàng một bài.

Bài 5: Cho $P\left( x \right)$ là một đa thức bậc $n \ge 1$ với hệ số thực và có $n$ nghiệm thực. Chứng minh rằng:
$$\left ( n-1 \right ){P}'^{2}\left ( x \right )\geq nP\left ( x \right ){P}''\left ( x \right ),\; \forall x\in \mathbb{R}$$


Giải
Do đa thức $P(x)$ là đa thức bậc $n$ có $n$ nghiệm nên đa thức có dạng
\[P(x) = a\left( {x - {x_1}} \right)\left( {x - {x_2}} \right)\left( {x - {x_3}} \right)....\left( {x - {x_n}} \right);(a \ne 0)\]
Vậy thì
\[P'(x) = P(x)\left( {\frac{1}{{x - {x_1}}} + \frac{1}{{x - {x_2}}} + ... + \frac{1}{{x - {x_n}}}} \right)\]
\[P''(x) = P'(x)\left( {\frac{1}{{x - {x_1}}} + \frac{1}{{x - {x_2}}} + ... + \frac{1}{{x - {x_n}}}} \right) - P(x)\left( {\frac{1}{{{{\left( {x - {x_1}} \right)}^2}}} + \frac{1}{{{{\left( {x - {x_2}} \right)}^2}}} + ... + \frac{1}{{{{\left( {x - {x_n}} \right)}^2}}}} \right)\]
Bất đẳng thức đã cho tương đương
\[\left( {n - 1} \right){\left( {\frac{1}{{x - {x_1}}} + \frac{1}{{x - {x_2}}} + ... + \frac{1}{{x - {x_n}}}} \right)^2} \ge n\left[ {{{\left( {\frac{1}{{x - {x_1}}} + \frac{1}{{x - {x_2}}} + ... + \frac{1}{{x - {x_n}}}} \right)}^2} - \left( {\frac{1}{{{{\left( {x - {x_1}} \right)}^2}}} + \frac{1}{{{{\left( {x - {x_2}} \right)}^2}}} + ... + \frac{1}{{{{\left( {x - {x_n}} \right)}^2}}}} \right)} \right]\]
Hay là
\[n\left( {\frac{1}{{{{\left( {x - {x_1}} \right)}^2}}} + \frac{1}{{{{\left( {x - {x_2}} \right)}^2}}} + ... + \frac{1}{{{{\left( {x - {x_n}} \right)}^2}}}} \right) \ge {\left( {\frac{1}{{x - {x_1}}} + \frac{1}{{x - {x_2}}} + ... + \frac{1}{{x - {x_n}}}} \right)^2}\]
Đúng theo bất đẳng thức Cauchy-Schwarz
@@Anh Thành:Cảm ơn anh đã ủng hộ topic của em mong anh giúp đỡ nhiều hơn :icon6:
alex_hoang


HẸN NGÀY TRỞ LẠI VMF THÂN MẾN

http://www.scribd.co...oi-Ban-Cung-The

#10
alex_hoang

alex_hoang

    Thượng úy

  • Hiệp sỹ
  • 1152 Bài viết
Bài 6:Cho đa thức $P(x)$ là đa thức với hệ số nguyên không âm và không lớn hơn $8$.Giả sử $P(9)=32078$.Hãy xác định đa thức $P(x)$
alex_hoang


HẸN NGÀY TRỞ LẠI VMF THÂN MẾN

http://www.scribd.co...oi-Ban-Cung-The

#11
alex_hoang

alex_hoang

    Thượng úy

  • Hiệp sỹ
  • 1152 Bài viết
Bài 7:Giả sử đa thức $P(x),Q(x),R(x)$ và $S(x)$ thỏa mãn đồng nhất thức
\[P({x^5}) + xQ({x^5}) + {x^2}R({x^5}) = \left( {{x^4} + {x^3} + {x^2} + 1} \right)S(x)\]
Chứng minh rằng đa thức $P(x)$ chia hết cho đa thức $x-1$
alex_hoang


HẸN NGÀY TRỞ LẠI VMF THÂN MẾN

http://www.scribd.co...oi-Ban-Cung-The

#12
alex_hoang

alex_hoang

    Thượng úy

  • Hiệp sỹ
  • 1152 Bài viết
Bài 8:Tìm tất cả các đa thức có hệ số thực sao cho
$P(2)=12$ và $P({x^ 2})={x^ 2}({x^ 2}+1)P(x)$
alex_hoang


HẸN NGÀY TRỞ LẠI VMF THÂN MẾN

http://www.scribd.co...oi-Ban-Cung-The

#13
alex_hoang

alex_hoang

    Thượng úy

  • Hiệp sỹ
  • 1152 Bài viết
Bài 9:CMR với mọi số nguyên dương $n$ thì đa thức sau không thể có nhiều hơn một nghiệm thực
\[P(x) = 1 + x + \frac{{{x^2}}}{{x!}} + ... + \frac{{{x^n}}}{{n!}}\]

@@Mọi người cùng gửi những bài toán hay về đa thứ mà mọi người biết lên để tham khảo nhé :icon6:

Bài viết đã được chỉnh sửa nội dung bởi alex_hoang: 26-11-2011 - 14:37

alex_hoang


HẸN NGÀY TRỞ LẠI VMF THÂN MẾN

http://www.scribd.co...oi-Ban-Cung-The

#14
nguyen thai phuc

nguyen thai phuc

    Sĩ quan

  • Thành viên
  • 430 Bài viết

Bài 9:CMR với mọi số nguyên dương $n$ thì đa thức sau không thể có nhiều hơn một nghiệm thực
\[P(x) = 1 + x + \dfrac{{{x^2}}}{{x!}} + ... + \dfrac{{{x^n}}}{{n!}}\]

@@Mọi người cùng gửi những bài toán hay về đa thứ mà mọi người biết lên để tham khảo nhé :icon6:

${{P}_{n}}\left( x \right)=\sum\limits_{k=0}^{n}{\dfrac{{{x}^{k}}}{k!}}\Rightarrow P_{n}^{'}\left( x \right)={{P}_{n-1}}\left( x \right)$
Ta chứng minh ${{P}_{2k}}\left( x \right)>0,{{P}_{2k+1}}\left( x \right)\uparrow $ bằng quy nạp
${{P}_{2k}}\left( x \right)>0\Rightarrow P_{2k+1}^{'}\left( x \right)={{P}_{2k}}\left( x \right)>0\Rightarrow {{P}_{2k+1}}\left( x \right)\uparrow $
$x_0$ là nghiệm duy nhất của ${{P}_{2k+1}}\left( x \right)$ thì $x>{{x}_{0}}\Rightarrow {{P}_{2k+2}}\left( x \right)\uparrow ,x\le {{x}_{0}}\Rightarrow {{P}_{2k+2}}\left( x \right)\downarrow $ mà
$P_{2k+2}^{{}}\left( {{x}_{0}} \right)={{P}_{2k+1}}\left( {{x}_{0}} \right)+\dfrac{x_{0}^{2k+2}}{\left( 2k+2 \right)!}>0$
=> dpcm
Hình đã gửi

#15
alex_hoang

alex_hoang

    Thượng úy

  • Hiệp sỹ
  • 1152 Bài viết
Cường nghe vẻ lại đói bài rồi nhỉ :icon6:
Bài tiấp nha
Bài 11:Cho tam thức bậc hai $P(x)={x^ 2}+px+q$Hãy tìm tất cả các đa thức bậc bốn $Q(x)$ có hệ số cao nhất bằng $1$ sao cho
$P(Q(x))=Q(P(x))$
alex_hoang


HẸN NGÀY TRỞ LẠI VMF THÂN MẾN

http://www.scribd.co...oi-Ban-Cung-The

#16
alex_hoang

alex_hoang

    Thượng úy

  • Hiệp sỹ
  • 1152 Bài viết
Bài 12:Đa thức $P(x)$ thỏa mãn điều kiện

\[\left\{ {\begin{array}{*{20}{c}}
{P(2006) = 2006!}\\
{xP(x - 1) = (x - 2006)P(x)}
\end{array}} \right.\]
Chứng minh rằng đa thức$f(x) = {P^2}(x) + 1$ bất khả quy
alex_hoang


HẸN NGÀY TRỞ LẠI VMF THÂN MẾN

http://www.scribd.co...oi-Ban-Cung-The

#17
Crystal

Crystal

    ANGRY BIRDS

  • Hiệp sỹ
  • 5534 Bài viết

Cường nghe vẻ lại đói bài rồi nhỉ :icon6:
Bài tiấp nha
Bài 11:Cho tam thức bậc hai $P(x)={x^ 2}+px+q$Hãy tìm tất cả các đa thức bậc bốn $Q(x)$ có hệ số cao nhất bằng $1$ sao cho
$P(Q(x))=Q(P(x))$


Xét $Q\left( x \right) = P\left( {P\left( x \right)} \right)$ thì $Q\left( x \right)$ thoả đề bài. Ta chứng minh đa thức này là duy nhất.

Giả sử ta có đa thức $H\left( x \right)$ thoả đề bài sao cho $P\left( {H\left( x \right)} \right) = H\left( {P\left( x \right)} \right)$

Đặt $R\left( x \right) = P\left( {P\left( x \right)} \right) - H\left( x \right)$ thì $\deg R < 4$ suy ra:
$$R\left( {P\left( x \right)} \right) = P\left( {P\left( {P\left( x \right)} \right)} \right) - H\left( {P\left( x \right)} \right) = P\left( {P\left( {P\left( x \right)} \right)} \right) - P\left( {H\left( x \right)} \right)$$
$$ = {\left[ {P\left( {P\left( x \right)} \right)} \right]^2} + pP\left( {P\left( x \right)} \right) + q - {\left( {H\left( x \right)} \right)^2} - pH\left( x \right) - q$$
$$ = \left[ {P{{\left( {P\left( x \right)} \right)}^2} - {{\left( {H\left( x \right)} \right)}^2}} \right] + p\left[ {\left( {P\left( {P\left( x \right)} \right)} \right) - H\left( x \right)} \right]$$
$$ = R\left( x \right)\left[ {P\left( {P\left( x \right)} \right) + H\left( x \right) + p} \right] \Rightarrow \deg R.\deg P = \deg R + 4 \Rightarrow \deg R = 4$$
Suy ra điều vô lí. Vậy $$Q\left( x \right) = P\left( {P\left( x \right)} \right) = {x^4} + 2p{x^3} + \left( {{p^2} + p + 2q} \right){x^2} + p\left( {2q + p} \right)x + q\left( {p + q + 1} \right)$$

#18
Crystal

Crystal

    ANGRY BIRDS

  • Hiệp sỹ
  • 5534 Bài viết

Bài 6:Cho đa thức $P(x)$ là đa thức với hệ số nguyên không âm và không lớn hơn $8$.Giả sử $P(9)=32078$.Hãy xác định đa thức $P(x)$


Giả sử đa thức cần tìm có dạng: $$P\left( x \right) = {a_n}{x^n} + {a_{n - 1}}{x^{n - 1}} + ... + {a_1}x + {a_0}$$
Theo giả thiết thì: $$P\left( 9 \right) = {a_n}{9^n} + {a_{n - 1}}{9^{n - 1}} + ... + {a_1}9 + {a_0} = 32078$$
Vì ${a_i},i = \overline {0,n} $ là số nguyên không âm nhỏ hơn 9 nên:
$$P\left( 9 \right) = {a_n}{9^n} + {a_{n - 1}}{9^{n - 1}} + ... + {a_1}9 + {a_0} = \overline {{a_n}{a_{n - 1}}...{a_0}_{\left( 9 \right)}} = 32078$$
Trong đó: $\overline {{a_n}{a_{n - 1}}...{a_0}_{\left( 9 \right)}} $ là biểu diễn của $32078$ trong hệ đếm cơ số $9$ và $32078 = {48002_{\left( 9 \right)}}$
Nên $$\overline {{a_n}{a_{n - 1}}...{a_0}_{\left( 9 \right)}} = 48002 \Rightarrow n = 4,{a_4} = 4,{a_3} = 8,{a_2} = 0,{a_1} = 0,{a_0} = 2$$
Suy ra đa thức cần tìm là: $P\left( x \right) = 4{x^4} + 8{x^3} + 2$. Thử lại thấy đúng.

#19
Crystal

Crystal

    ANGRY BIRDS

  • Hiệp sỹ
  • 5534 Bài viết
Bài 13: Cho $P\left( x \right)$ là đa thức khác không, $\deg P < 2012$ và không có thừa số chung với ${x^3} - x$. Giả sử ${\left( {\frac{{P\left( x \right)}}{{{x^3} - x}}} \right)^{\left( {2009} \right)}} = \frac{{Q\left( x \right)}}{{R\left( x \right)}}$, trong đó $Q\left( x \right),\,R\left( x \right)$ là những đa thức nào đó. Chứng minh rằng $\deg Q\left( x \right) \ge 4018$.

Bài viết đã được chỉnh sửa nội dung bởi xusinst: 27-11-2011 - 11:50


#20
alex_hoang

alex_hoang

    Thượng úy

  • Hiệp sỹ
  • 1152 Bài viết
Trong khi chờ bài 13 của anh Thành được sửa đề thì ta làm mấy bài này cho đỡ buồn anh em nhé
Bài 14:Cho đa thức $f(x)$ là một đa thức có hệ số thực thỏa mãn điều kiện
\[\left\{ {\begin{array}{*{20}{c}}
{0 < {f^2}(xy) \le f(x).f({y^3}) + f({x^3}).f(y)}\\
{f(2007) > 0}
\end{array}} \right.\]
Chứng minh rằng $f(2008)>0$
alex_hoang


HẸN NGÀY TRỞ LẠI VMF THÂN MẾN

http://www.scribd.co...oi-Ban-Cung-The




0 người đang xem chủ đề

0 thành viên, 0 khách, 0 thành viên ẩn danh